Adult Health 1 Exam #2

Ace your homework & exams now with Quizwiz!

What procedure used in the treatment of osteoarthritis involves removing a small amount of bone at the articulating surface of the joint and fitting a metal replacement over the end of the bone?

joint resurfacing

Bloom's Taxonomy

knowledge, comprehension, application, analysis, synthesis, evaluation

Metamucil

laxative -Indicated for treatment of constipation -increases bulk in stool; increases water in stool (makes stools softer) -may cause gas, abdominal cramping

Hypovolemia

loss of both water and electrolytes in similar proportions from ECF

CBC

complete blood count

Factors influencing urinary patterns

-Disease conditions -Privacy -Psychological -Fluids -Surgery -Medications -Age -Food -Muscle tone -Dx evaluation -Tea, Alcohol, Caffiene -Inability to access bathroom -Decreased bladder capacity

A patient reports experiencing headaches in the morning. Which question should the nurse ask the patient?

"How many hours a night do you sleep?" Rationale:Manifestations of sleep disorders vary from sleeplessness and fatigue to irritability, distractibility, and morning headaches. The patient's headaches occur in the morning, so the other questions can be asked after the patient's sleep pattern has been established.

The nurse is providing discharge teaching to a patient diagnosed with urinary incontinence. Which patient statement indicates the need for further teaching regarding preventive methods for urinary incontinence?

"I have switched to a low-fiber diet." Rationale:A low-fiber diet is not indicated as a preventive method of decreasing urinary incontinence. The other patient statements indicate understanding of the teaching session.

The nurse who is caring for an older adult at risk for osteoporosis discusses the importance of weight-bearing activity. Which statement made by the patient requires further teaching?

"I enjoy swimming, so I will try and swim at least 3 times a week." Rationale: The patient statement, "I enjoy swimming, so I will try and swim at least 3 times a week," requires further teaching. Swimming is not a weight-bearing exercise. Weight-bearing exercises, such as tai chi, weight lifting, and walking on a golf course, influence bone metabolism in several ways. The stress of this type of exercise causes an increase in blood flow to bones, which brings growth-producing nutrients to the cells, resulting in increased osteoblast growth and activity.

Magnesium

1.5-2.5 mg/dL

BUN

6-20 mg/dL

pH

7.35-7.45

Colloid Osmotic Pressure

A pulling force exerted by colloids that helps maintain the water content of blood by pulling water from the interstitial space into the vascular compartment

Nicardipine (Cardene)

Antihypertensive-Calcium Channel Blocker -Indicated for chronic/stable angina, HTN -May cause headache, dizziness, angina, nausea, vomiting -Decreases smooth muscle contractility in the bladder

A patient in the clinic provides a urine sample that is red-orange in color. What action should the nurse take?

Ask the patient about current medications

Most common side effect of using a CPAP

Dry mouth

physical factors affecting mobility

Inflammatory- rheumatoid arthritis Degenerative - osteoarthritis Traumatic - fractures and dislocations Congenital - occur at birth

Effects of Sleep Disorders

Neuro: Cognitive impairments, behavioral changes Immune: impaired function Respiratory: Asthma exacerbated by sleep Cardiac: Heart Disease, increased BP, Stroke GI: Obesity, GERD Endocrine: Type 2 Diabetes, insulin resistance, Growth hormone

hypertonic

Refers to solutions that have a higher osmolarity than body fluids

What term is used to describe osteoarthritis that is caused by an underlying condition, such as injury, congenital malformation, or metabolic disease?

Secondary

Restless Leg Syndrome

a neurologic sensorimotor disorder that is characterized by an overwhelming urge to move the legs when at rest

polysomnography

a recording of the biophysical changes that a patient experiences during sleep

imagery

a relaxation technique in which the patient focuses on pleasant images such as a beach or a garden to replace negative images such as pain and darkness

Hypermagnesemia

an excess of magnesium in the blood plasma

Flatus

gas or air normally present in the stomach or intestines

isometric exercise

static or sitting exercises in which muscles contract without moving the joint

Renal Calculi

stones in the kidney

Acute pain

temporary, localized, and sudden pain that lasts for less than 6 months and has an identifiable cause, such as trauma, surgery, or inflammation

circadian rhythm

the biological clock; regular bodily rhythms (for example, of temperature and wakefulness) that occur on a 24-hour cycle

Range of Motion (ROM)

the degree to which a joint is able to move; measurement of flexion and extension

glycosuria

the excretion of carbohydrates into the urine

urinalysis (UA)

urine screening test that includes physical observation, chemical tests, and microscopic evaluation

Residual Urine

urine that remains in the bladder after voiding

Nocturia

voiding 2 or more times at night

The nurse concludes that a client is demonstrating cardiovascular manifestations of undernutrition. Which assessment finding supports this​ conclusion?

B. Irregular heartbeat E. Low blood pressure

Major complication of immobility

damage to the skin from prolonged pressure on bony prominences - pressure injuries *patients will need to be laid in bed in an anatomical position

Hyponatremia

decrease blood levels of sodium

Hyperkalemia

excessive potassium in the blood

Hypervolemia

excessive retention of water and sodium in ECF

Constipation

fewer than 3 bowel movements per week or the difficult passage of stools

interstitial fluid

fluid between cells

Extracellular Fluid

fluid found outside the body cells

intravascular fluid

fluid within blood vessels

Intracellular fluid (ICF)

fluid within cells

An ambulatory elderly woman with dementia is incontinent of urine. She has poor short term memory and has not been seen toileting independently. What is the best nursing intervention for this patient?

Start a scheduled toileting program

Joint Deformity

Distortion or permanent deviation of normal structures of joints

Anuria

Failure of the kidneys to produce urine, resulting in a total lack of urination or output of less than 100 mL/day in an adult

What are some causes of C. Diff?

1. Contact with C. Diff bacteria 2. Overuse of antibiotics

Modifiable Risks Factors for Mobility

-Weight (obesity a major risk factor) -Well-balanced diet

Hypoxia

Decreased delivery of oxygen to the tissues

Which of the following is a common element of abuse experienced by the​ victim?

Humiliation

Tender points

Tenderness that occurs in precise, localized areas, particularly in the neck, spine, shoulders, and hips.

activities of daily living (ADLs)

basic self-care tasks such as eating, bathing, toileting, walking, and dressing

Bruits

blowing sound sometimes heard due to restriction of blood flow through the vessels

The nurse is caring for a client with osteoarthritis. What factor in the client's history and physical assessment would the nurse recognize as a risk factor for developing this condition?

body mass index of 36.5

How do anticholinergics cause urinary retention?

by decreasing bladder detrusor muscle contraction

Bowel Incontinence (Fecal Incontinence)

inability to control the discharge of feces and flatulence

Milk of Magnesia (MOM)

osmotic laxative -indicated for treatment of occasional constipation -used for a short time -draws water into the intestines that helps cause movement in the intestines -may cause diarrhea

Ditropan

oxybutynin/functional incontinence med -increased bladder capacity and decreases frequency of voiding in clients with neurogenic bladder -side effects: dry mouth, drowsiness, blurred vision, constipation, and urinary retention, angle narrow glaucoma

fleets enema

saline laxative, route-rectal -indicated for treatment of occasional constipation, used to clean intestines pre-operatively -may cause mild abdominal discomfort, cramps, gas

nocturia

voiding two or more times at night

osteoporosis risk factors

white and asian american women have a higher incidence of osteoporosis than native american, hispanic, and african american women -risk of fracture in white women is higher than in non-white women of the same age

POLICE acronym

Protection Optimal Load Ice Compression Elevation

Synovium

Soft tissue that lines the spaces of diarthroidal joints, tendons, and bursae

What information with the nurse include when teaching a patient how to avoid chronic constipation?

1. Bulk-forming laxatives are an excellent source of fiber. 2. Walking or cycling frequently will help bowel motility 3. A good time for a bowel movement may be after breakfast 4. Some over the counter medications cause constipation

Sodium

135-145 mg/dL

HCo3

22-26

potassium

3.5-5.0 mg/dL

PaCO2

35-45

Epinephrine (Epipen)

Adrenergic/Vasopressor -Indicated for anaphylaxis, hypotension associated with shock, asthma, cardiac resuscitation -May cause drowsiness, headache, palpitations, shock, tachycardia, nausea, vomiting, dyspnea -Drug interferes with lab specimens that tests for urinary retention

Non-Rapid Eye Movement (NREM) Sleep

occurs when activity in the reticular system is inhibited

A 76 year old patient with constipation has fecal impaction and is incontinent of liquid stool. What action should the nurse take first?

Manually remove the impacted stool

Diuretics

Pharmacologic agents that increase urine formation and secretion

Sprains and Stains RICE Therapy

Rest Ice Compression Elevation

Fibercon

treatment of constipation, bulk forming laxative -increases the amount of water in the stool, making the stool softer -may cause gas, stomach cramping

Hypomagnesemia

decreased blood levels of magnesium

comfort

to ease the grief or trouble of others; to give hope

The nurse is assessing a client for physical problems that affect food intake. Which factor should the nurse​ consider?

A. Trouble swallowing D. Problems with dentition

What procedure would be most appropriate to repair a finger joint that is affected by severe osteoarthritis?

Joint fusion

A patient who has chronic constipation asks the nurse about the use of psyllium (metamucil). What information will the nurse include in the response

Large amounts of fluid should be taken to preent impaction or bowel obstruction

Causes of Osteoarthritis

trauma, sprains, strains, joint dislocation, fractures, mechanical stress, neurologic disorders, drugs

The nurse is caring for an older adult who has advanced dementia, osteoporosis, and frequently gets out of bed throughout the night. Which nursing intervention is most appropriate for the nurse to include in the plan of care?

Placing the bed in the lowest position Rationale: The safest nursing intervention to prevent injury to the patient with advanced dementia who frequently gets out of bed at night is to place the bed in the lowest position. Medicating the patient is a chemical restraint. Providing the patient with an assistive device is necessary if one is needed and the patient can use it safely. Restraints should be avoided, because they may actually increase the patient's risk for falling and the risk of injury associated with the fall.

Hydrostatic Pressure

Pressure exerted by a volume of fluid against a wall, membrane, or some other structure that encloses the fluid.

relaxation techniques

Reduce stress by slowing the heart and respiratory rates, lowering blood pressure, and increasing blood flow to major muscles. 4 major categories of relaxation techniques include: 1. breathing. 2 muscle relaxation. 3. imagery. 4. movement

motofen (difenoxin/atropine)

antidiarrheal -treatment of diarrhea -works by slowing the movement of the intestines -may cause drowsiness, headache, dry mouth, loss of appetite -do not use in C. Diff patients

common sites of osteoarthritis

cervical vertebrae, lower lumbar vertebrae, hip, metacarpophalangeal, proximal interphalangeal, distal interphalangeal, knee, metatarsophalengeal

Embolus

clot of material that travels through the bloodstream and suddenly blocks a vessel

Hypernatremia

elevated blood sodium level

Hypercalcemia

excessive calcium in the blood

When a patient has fecal incontinence as a result of cognitive impairment, it may be helpful to teach caregivers to do what intervention?

initiate bowel or habit training program to promote incontinence

Extracellular fluid volume deficit

insufficient isotonic fluid in the extracellular compartment; dehydration

sleep hygiene

interventions used to promote sleep

Creatinine

0.6-1.3 mg/dL

The nurse is providing care for an older adult female patient who states, "I have been experiencing low back pain, which has been causing me to lose sleep." Which question will best help the nurse determine if the patient's pain is associated with osteoporosis?

"Do you have a history of fractures?" Rationale:The question the nurse will ask the patient that is related to osteoporosis is, "Do you have a history of fractures?" A history of fractures and low back pain are risk factors for osteoporosis. Decreased range of motion, arthritis, and position of sleep are not directly related to osteoporosis.

A client with osteoarthritis of the knees and hips returns for a 3 month follow up with the provider. The nurse calculates that the clients body mass index (BMI) is now 22. The client reports starting a water aerobics and running program 3 times a week. The client is also using hot packs for edema for 20 minutes and cold packs for 40 minutes daily. After evaluating the clients actions, what follow up interventions should the nurse plan?

1. Congratulate the client on starting water aerobics 2. Reinforce the correct use of hot packs 3. Suggest the client replace running with a lower impact exercise 4. Explain the risk of injury associated with use of cold packs

What are key points that the nurse should include in patient education for a person with complaints of chronic constipation?

1. Increase fiber and fluids in the diet 2. Exercise for 30 minutes every day. 3. Schedule time to use the toilet at the same time every day.

symptoms of osteoarthritis

1. Pain (w/ or w/o motion) 2. Morning stiffness less than 15 min 3. Limited range of motion (ROM) 4. Crepitus (cracking sounds in joints) 5. Instability 6. Joint Tenderness 7. Muscle atrophy

Glomerular Filtration Rate (GFR)

90-120 mL/min the rate at which fluid is filtered through the kidneys

The nurse reviews the universal aspects of comfort with a group of staff members. Which statement should the nurse​ include? (Select all that​ apply.) A. Giving and receiving respect are aspects of​ self-esteem needs. B. Emotional needs include love and belonging from family and friends. C. Emotional needs are higher priority than are physiological needs. D. Physiological needs include​ oxygen, shelter,​ food, water, and sleep. E.When physiological needs are​ met, other needs can be achieved.

A. Giving and receiving respect are aspects of self-esteem needs B. Emotional needs include love and belonging from family and friends D. Physiological needs include oxygen, shelter, food, water, and sleep E. When physiological needs are met, other needs can be acheived.

The nurse is teaching a group of staff members about parasomnias. Which example should the nurse use when explaining this​ disorder? (Select all that​ apply.) A. Night terrors B. Narcolepsy C. Bruxism D. Enuresis E. Sleep talking

A. Night terrors C. Bruxism D. Enuresis E. Sleep talking

What treatment option would least likely be considered for a 71 year old client with osteoarthritis?

Administration of NSAIDs

A patient diagnosed with osteoporosis states to the nurse, "I don't understand how my bones can be so brittle and break easily." Before responding to the patient, the nurse should understand that which process is involved in the pathophysiology of osteoporosis?

An imbalance between osteoblasts and osteoclasts has occurred. Rationale: Osteoporosis is a metabolic bone disorder characterized by loss of bone mass, increased bone fragility, and increased risk of fractures. Although the exact pathophysiology of osteoporosis is unclear, it is known to involve an imbalance in the activity of osteoblasts that form new bone and osteoclasts that reabsorb bone. Osteoclasts are responsible for reabsorption of bone and osteoblasts the formation of new bone.

Which condition should the nurse recognize as a cause of fecal incontinence?

Anorectal Injury Rationale:Irritable bowel syndrome may cause diarrhea, but not necessarily incontinence. If the external anal sphincter is paralyzed by injury or disease (anorectal injury), defecation occurs automatically when the internal sphincter relaxes. Irregular defecation habits increase the patient's risk for constipation, not fecal incontinence. Gastrointestinal reflux disease is the regurgitation of stomach acids back into the esophagus. It does not cause fecal incontinence.

Glycopyrronium

Anticholinergic -Treatment of abdominal pain, peptic ulcer -Slows natural movement of the gut -may cause dizziness, dry mouth, constipation, and abdominal bloating

A patient has a history of a neurogenic bladder as a result of a spinal cord injury. What action will the nurse place to take first?

Ask about the usual urinary pattern and any measures used for bladder control

A patient calls the clinic complaining of diarrhea for 24 hours. What action should the nurse take first?

Ask the patient to describe the character of the stools and any associated symptoms

A nurse is teaching the parents of a client who was recently diagnosed with osteoarthritis about their childs condition. What statement by the parents indicates the need for *further* education?

Because our daughter developed OA as a child, she is more likely to become disable as a result of this condition.

functional incontinence

Loss of urine resulting from cognitive, functional, or environmental factors

REM sleep

Rapid eye movement sleep, a recurring sleep stage during which vivid dreams commonly occur. Also known as paradoxical sleep, because the muscles are relaxed (except for minor twitches) but other body systems are active.

Pain Scale

assessment tool used to rate the severity of pain

orthostatic hypotension

low blood pressure that occurs upon standing up

Factors affecting urinary patterns

-bladder irritation (infection, trauma) -impaired ability to recognize bladder -weak pelvic flood muscles -bladder outlet obstruction -dehydration -diagnostic procedures -anesthesia/surgery -pathologic conditions

Developmental factors that can alter mobility in older adults

-muscle weakness -atrophy -osteoarthritis

Toxicology screening

-performed in emergency situations -can be done to monitor drug dependency, substance abuse, or part of job requirements -screening conducted via: urine drug screen, venipuncture, gastric lavage

Osteoporosis

-porous bone -metabolic bone disorder characterized by loss of bone mass, increased bone fragility, and increased risk of fractures -inadequate calcium intake

Enema

-the placement of a solution into the rectum and colon to empty the lower intestine through bowel activity -Fleets (saline, mineral oil)

What is an example of chronic​ pain? A. Pain that outlasts the healing process B. Pain that follows injury and ends when healing is complete C. Pain that precedes injury D. Pain that is felt during injury and immediately after

A. Pain that outlasts the healing process

Which nursing intervention should be implemented for a client at risk of nutritional​ imbalances?

A. Providing nutritional education B. Promoting healthy eating habits

Hypnotics

drugs used to induce sleep

Which statement regarding comfort is true? Select All That Apply: A. It can be assessed by vital signs B. It varies from one individual to another C. It is subjective D. It is objective E. It can be associated with sleep and rest

B. It varies from one individual to another C. It is subjective E. It can be associated with sleep and rest

Which factor increases an​ individual's risk for experiencing obstructive sleep​ apnea? (Select all that​ apply.) A. Female gender B. Large neck circumference C. Narrow airway D. Smoking E. Obesity

B. Large neck circumference C. Narrow airway D. Smoking E. Obesity

Polydipsia

excessive thirst

Defecation

expulsion of feces from the anus and rectum

neglect

fail to care for properly

borborygmus

hyperactive, high-pitched, tinkling, rushing, or growling bowel sounds heard in diarrhea or at the onset of bowel obstruction

Chronic pain

prolonged pain, usually lasting longer than 6 months. It is not always associated with an identifiable cause and is often unresponsive to conventional medical treatment

Clients with osteoarthritis can reduce their risk of further joint damage by doing what?

receiving cortisone injections in affected joints no more than 3 times per year

Aspirin

-NSAID -Indicated for rheumatoid arthritis, osteoarthritis, mild pain, fever, suspected acute MI, reduce risk of recurrent TIAs -May cause GI distress, bleeding, and allergic reactions

Dutasteride (Avodart)

BPH - 5 alpha reductase inhibitor -Indicated for treatment and improvement of symptoms of BPH, reduce risk of acute urine retention, and reduce the need for BPH related surgery

Tamsulosin (Flomax)

BPH alpha blocker -Treatment of urinary retention, treatment of ureteral stones -May cause dizziness, headache, chest pain, diarrhea, nausea, decreased libido, back pain, infection -Monitor patient for decreased blood pressure -Take med 30 minutes after meal each day

The nurse is teaching a patient who is diagnosed with osteoarthritis about exercise. Which recommendation should the nurse include?

Balance and agility exercises can help maintain daily living skills." Rationale:Balance and agility exercises are recommended for patients with osteoarthritis because they help to maintain daily living skills. When beginning strengthening exercises, patients should start with a low weight and work their way up. Water exercise is beneficial because the buoyancy of the water decreases the force on the joints. The patient should stretch all muscle groups for 10 minutes each day. Overstretching is contraindicated.

Terbutaline

Beta 2 agonist/Bronchodilator -Indicated for prevention/reversal of bronchodilator -May cause nervousness, tremor, drowsiness, palpitations, nausea/vomiting, hypokalemia, dyspnea -Off label use used in the treatment of bladder hyperactivity, stress, and overflow incontinence

A public health nurse is presenting a teaching session about alcohol use to a group of college seniors. During the​ session, one of the students admits to frequent alcohol use. What is the​ nurse's priority​ action?

Complete a crisis assessment with the student

Muscle Tone

Degree of muscle tension or resistance during rest or in response to stretching.

Dysuria

Difficult or painful urination

Peri-Colace

Emollient Laxative -Indicated for treatment of constipation -may cause abdominal pain, cramping, nausea, diarrhea, weakness

Five common sources of discomfort

Pain, end-of-life care, fatigue, fibromyalgia, and sleep-rest disorders.

Factors affecting bowel elimination

age, diet/fluid intake, physical activity, psychological factors, personal habits, position during defecation, pregnancy, surgery/ anesthesia, meds/laxatives/ cathartics

Factors affecting bowel elimination

age, diet/fluid intake, physical activity, psychological factors, personal habits, position during defecation, pregnancy, surgery/ anesthesia, meds/laxatives/ cathartics/pain

Dicyclomine (Bentyl)

antispasmodic, anticholinergic -Treatment of IBS -May cause headache, dizziness, drowsiness, palpitations, constipation, dry mouth, urine retention/hesitancy, allergic reactions

sleep deprivation

any significant loss of sleep, resulting in problems in concentration and irritability

BMP

basic metabolic panel

Hypocalcemia

deficient calcium in the blood

Hypokalemia

deficient potassium in the blood

Hyponatremia

deficient sodium in the blood

drug abuse

intentionally using drugs in a way that is unhealthy or illegal

neurogenic bladder

interference with the normal mechanisms of urine elimination in which the client does not perceive bladder fullness and is unable to control the urinary sphincters; the result of impaired neurologic function

biological clocks

internal timing devices that are genetically set to regulate various physiological responses for different periods of time

urge incontinence

involuntary loss of urine associated with a strong urge to void, increased rate of urination, inability to avoid urinating until a bathroom is available, overactive detrusor muscle, and nocturia

Crystalloids

salts that dissolve readily into true solutions

Hypersomnolence Disorder

sleep dysfunction involving an excessive amount of sleep that disrupts normal routines

NREM sleep

slow eye movement heart rate and temperature decreased deep or slow wave sleep (delta waves)

Smoking abuse

smoking too much, too often, or at inappropriate times

Detrusor Muscle

smooth muscle layers of the bladder wall; allows the bladder to expand as it fills with urine and contract as it releases urine during voiding.

Peristalsis

the process of wave-like muscle contractions that propels food and digestive products through the digestive tract

An older adult patient taking prescribed medication for restless leg syndrome expresses the desire to use an herbal tea to improve sleep. How should the nurse respond to this patient?

"Certain herbal supplements may interact with prescribed medications, so it's always best to check with your healthcare provider first." Rationale:Because herbal supplements may interact with prescribed sleep aids, the patient should consult the healthcare provider before adding herbal agents to the treatment regimen. As with prescribed medications, certain herbal supplements may be contraindicated for specific patients, regardless of whether the herbal supplement is natural. Although chamomile and valerian sometimes are used by patients for the treatment of certain alterations in sleep-rest patterns, the effectiveness of these herbal supplements has not been clinically proven, and they can interact with other medications.

A patient who reports difficulty falling asleep is slumped over, has dark circles under the eyes, and is slow to respond to questions. Which question should the nurse ask this patient?

"Do you drink caffeinated beverages or alcohol?" Rationale:The patient complaining of fatigue and an inability to fall asleep should be screened for insomnia. Alcohol and caffeine can contribute to sleep-rest disorders such as insomnia. When asking a question about exercise, the information centers around how often, how long, what type, and the time of day the patient exercises. Asking the patient if anything unusual happens when the patient laughs or gets angry is an assessment for cataplexy that is associated with narcolepsy. The nurse will assess how long the patient generally sleeps each night to get an overview. The assessment of one night of sleep does not provide adequate information on which to base judgment.

A patient with osteoarthritis reports poor pain control. Which patient statement indicates a need for follow-up by the nurse?

"I make sure to take my acetaminophen at the first sign of pain." Rationale:To most effectively prevent severe pain related to osteoarthritis (OA), the patient should routinely take the acetaminophen, so this statement would require further explanation and clarification. Use of hot packs for 20 minutes, range-of-motion exercises of all joints daily, and resting the affected joints when increased pain occurs are all appropriate activities for managing OA.

A patient does not want to use the continuous positive airway pressure (CPAP) machine because of discomfort and waking up with a dry mouth. How should the nurse respond to this patient?

"I recommend you use an inline or room humidifier." Rationale:The most common side effect of CPAP or bilevel positive airway pressure (BiPAP) therapy is a dry mouth and airway, so an inline or room humidifier is recommended. A mask that does not sit right will not cause a dry mouth. Adjusting the air pressure of the machine will minimize difficulty exhaling. Drinking plenty of fluid while using a CPAP mask will not remedy the dryness caused by the use of a CPAP machine.

The nurse teaches a client with a sleep disturbance about sleep hygiene. Which patient statement indicates that additional teaching is required?

"I will only go to bed when I am tired." Rationale:The patient should maintain a consistent bedtime. The overall goal is for the patient to sleep through the night, feel more energetic throughout the day to maintain normal daily activities, and use relaxation techniques 30-45 minutes prior to bedtime.

The nurse has completed the medication teaching for the patient prescribed a bisphosphonate for osteoporosis. Which patient statement indicates that further teaching is required?

"I will take my medication with my breakfast, so I do not get nauseated." Rationale:The statement "I will take my medication with my breakfast, so I do not get nauseated" indicates that further teaching is required. Bisphosphonates should be taken on an empty stomach with water first thing in the morning. The patient should remain upright for 30 minutes and should not eat or drink anything else for 30 minutes to avoid esophagitis. A dietary log is helpful to the healthcare provider and the patient to ensure that adequate dietary calcium, vitamin D, and phosphate are consumed. Walking is a weight-bearing exercise that is important in the prevention of further complications of osteoporosis. Calcium and vitamin D supplements should be held 60 minutes or longer after taking the bisphosphonate.

A patient who is scheduled for a polysomnography (PSG) asks the nurse to explain the purpose of using video and audio equipment during the sleep study. How should the nurse respond to this patient?

"It may be useful for detecting various sleep disorders." Rationale:Monitoring with video and audio equipment may be useful for detecting parasomnias (sleep disorders) such as sleepwalking, sleep talking, and night terrors. Leg movements are monitored to detect periodic limb movement disorder and restless leg syndrome. PSG testing also includes electroencephalogram (EEG) monitoring to evaluate the brain waves associated with non-REM (NREM) and REM sleep patterns, which can be analyzed to identify sleep disorders.

A patient is being scheduled for an osteotomy and asks the nurse what the surgery entails. Which response by the nurse is accurate?

"It realigns or shifts the joint load into a better aligned point or toward a less damaged part of the joint." Rationale:An osteotomy realigns or shifts the joint. A joint fusion, not an osteotomy, fuses two bones together and may use bone grafts. Arthroscopy is used to determine the extent of joint damage in preparation for future surgery.

A patient who works the night shift is unable to sleep during the day and asks for a natural substance to promote sleep. How should the nurse respond to this patient?

"Melatonin is the best option because it is a hormone your body already produces." Rationale:Melatonin is a sleep hormone produced by the pineal gland. Synthetic melatonin is sold in many pharmacies and health food stores and may be taken to regulate sleep patterns. It is often helpful for sleep disturbances related to shift work or jet lag. St. John's wort is thought to be effective in improving mood and decreasing the anxiety and insomnia that occur in relation to depression. Chamomile tea has a soothing effect that may induce sleep and decrease restlessness, although this effect has not been proven in studies. Valerian has been used as a sleep aid and usually has to be taken for 2 or 3 weeks before it produces an effect. Clinical trials have not proven its effectiveness.

A patient who recently learned of the diagnosis of narcolepsy asks what causes the disorder. Which response should the nurse make?

"Narcolepsy is sometimes linked to a deficiency of a brain neurotransmitter." Rationale:Narcolepsy, which is a severe form of hypersomnia, is a nervous system disorder where the brain is unable to regulate sleep-wake cycles. Sometimes it is linked to a deficiency in hypocretin, a brain neurotransmitter. Obstructive sleep apnea occurs when the airway becomes blocked by the soft palate, tongue, and uvula. Central sleep apnea occurs due to temporary failure of the chest muscles and diaphragm. Getting less than 7-8 hours of sleep each night can result in sleep loss.

Which recommendation should the nurse include when teaching guidelines to maintain a healthy body mass index?

"Participate in regular exercise, including walking or swimming." Rationale:Keeping weight down to a healthy level can be achieved by participating in regular low-impact aerobic exercises, such as walking, cycling, or swimming. These are well tolerated by patients with osteoarthritis (OA). These exercises can also keep joints strong and functional. Heavy weight lifting is not recommended for a patient with OA and may actually lead to further joint breakdown. Exercise is proven to prevent OA; it should not be discouraged. If pain is experienced with exercise, the patient should stop and rest. Continuing may build muscle but will break down the joint cartilage.

A middle-aged patient reports frequent nighttime waking despite being informed by the spouse of snoring throughout the night while asleep. Which response should the nurse make to this patient?

"Snoring may be a symptom of airway obstruction, which could cause some problems with the quality of your sleep." Rationale:Although snoring does not always indicate that apnea is occurring, snoring is a primary symptom of obstructive sleep apnea, which occurs when the airway becomes blocked by the soft palate, tongue, and uvula. A patient's spouse or partner is a helpful source of information about the patient's sleep habits. The patient does not need to record snoring in order to validate the spouse's report. Snoring may be indicative of obstructive sleep apnea, and the issue should be investigated regardless of whether the patient's snoring interrupts the spouse's sleep. Dyssomnia is also referred to as restless leg syndrome and is characterized by the need to move the legs when at rest.

The clinical nurse specialist is teaching a group of staff nurses about surgical treatments that may be included in the care of patients who are diagnosed with obstructive sleep apnea. Which statement should the nurse include?

"Surgery is usually only considered if continuous positive airway pressure (CPAP) therapy is not tolerated." Rationale:For patients with obstructive sleep apnea, surgical treatment may be an option, especially if the obstruction is caused by the tonsils (tonsillectomy) or adenoids (adenoidectomy). However, surgery is usually only considered if CPAP therapy is not tolerated. In children, an adenotonsillectomy may be performed to treat obstructive sleep apnea. Another procedure that may be performed to help treat obstructive sleep apnea is partial surgical removal of the soft palate, uvula, and posterior lateral pharyngeal wall (uvulopalatopharyngoplasty [UPPP]).

The nurse is providing care to a patient diagnosed with osteoporosis. Dual-energy x-ray absorptiometry (DEXA) has been ordered for the patient. Which explanation of this diagnostic procedure is appropriate for the nurse to give the patient?

"The test measures bone density in the lumbar spine or hip." Rationale: The DEXA measures bone density in the lumbar spine or hip. An ultrasound is administered to the heel of the foot to measure bone density. X-rays detect osteopenia and identify fractures. A quantitative ultrasound (QUS) is used to evaluate bone mineral density and the degree of osteoporosis.

A patient prescribed zolpidem (Ambien) for insomnia asks how long the medication needs to be taken. How should the nurse respond to this patient?

"This medication is for short-term use only." Rationale:Zolpidem is an anxiolytic that will induce tolerance over time. Long-term use is not recommended. The time necessary to diagnose the cause of the patient's insomnia will dictate the type of medication used to treat the insomnia. The patient will not become addicted to zolpidem in 1 week.

A patient diagnosed with osteoporosis asks, "How can I prevent this disease from progressing?" Which response by the nurse provides the patient with important dietary information to prevent the osteoporosis from progressing?

"To help prevent further progression of the disease, it is important for you to increase your calcium intake." Rationale:Calcium is an essential mineral in the process of bone formation and other significant body functions. When the intake of calcium through the diet is insufficient, the body compensates by removing calcium from the skeleton, weakening the bone tissue. The nurse should also remind the patient that vitamin D helps with calcium absorption. Foods high in vitamin A, iron, animal protein, and zinc are not effective in the prevention of the progression of osteoporosis.

A patient asks the nurse, "How does exercise prevent osteoporosis?" Which statement accurately describes the nurse's understanding of how weight-bearing exercise can help prevent osteoporosis?

"Weight-bearing exercise promotes osteoblast growth and activity." Rationale:Weight-bearing exercises, such as walking, influence bone metabolism in several ways. The stress of this type of exercise causes an increase in blood flow to the bones, which brings growth-producing nutrients to the cells. Walking causes an increase in osteoblast growth and activity. Osteoclast activity is the breaking down of the bone cells. Weight-bearing exercise keeps the calcium in the bones. Muscle strength is important to maintain activity, but it is not the primary reason that weight-bearing exercise is recommended to prevent osteoporosis.

The nurse is assessing a patient to elicit possible risk factors for the development of osteoarthritis. Which assessment question should the nurse ask?

"What type of work do you do?" Rationale:During the assessment, the nurse should ask the patient about their occupation since occupations that require repetitive-motion activities can increase the risk for osteoarthritis (OA). Alcohol, smoking, and supplements have not been implicated as risk factors for OA.

A patient is demonstrating symptoms of insomnia. Which question should the nurse ask this patient when completing the health history?

"When you fall asleep, are you able to remain asleep? Rationale: A clinical manifestation of insomnia is the inability to fall asleep or remain asleep. Experiencing frequent nighttime awakenings, headaches, and snoring is associated with obstructive sleep apnea.

Systemic Effects of Immobility

*Metabolic* Endocrine, calcium absorption, and GI function *Cardiovascular* Orthostatic Hypotension, thrombus *Muscle Effects* Loss of muscle mass, muscle atrophy *Urinary Elimination* Urinary stasis, renal calculi, infection *Respiratory* Atelectasis and hypostatic pneumonia *Musculoskeletal changes* loss of endurance and muscle mass and decreased stability and balance *Skeletal Effects* impaired calcium absorption, joint abnormalities *Integumentary* pressure ulcer, ischemia

Signs/Symptoms of Osteoporosis

- Back Pain - Kyphosis & scoliosis - Decreased height - Spontaneous Fractures - Compression fractures

CPAP and side effects

- Hypotension - Pneumothorax - Increased risk of aspiration - Drying of corneas -Dry mouth

Albuterol (Preventil)

-Adrenergic B2/Bronchodilator -Indicated for treatment of bronchospasm -Can cause urinary retention -May cause tremor, headache, dizziness, tachycardia, palpitations, nausea, vomiting

Tramadol (Ultram)

-Analgesic -Controlled substance type IV -Indicated for moderate to moderately severe chronic pain -May cause dizziness, headache, confusion, vasodilation, HTN, visual disturbances, constipation, nausea, vomiting, dry mouth, weight loss, arthralgia, respiratory depression, chills, withdrawl syndrome, accidental injury, addiction/abuse

Acetaminophen (Tylenol)

-Analgesic -Indicated for mild pain or fever; mild to moderate pain with adjunctive opioid analgesics -May cause nausea, vomiting, nausea, vomiting, abdominal pain, drug toxicity over 4000mg, dyspnea, hypoxia, anxiety, fatigue, headache, rash, urticaria

Calcium Gluconate

-Antidote - Calcium Salt -Prevent or treatment of low blood calcium levels, treat condition caused by low calcium levels such as bone loss (osteoporosis) -may cause constipation, gi distress, allergic reactions

Clonidine

-Antihypertensive/Alpha Agonist -Indicated for essential and renal hypertension, severe cancer pain, and ADHD for monotherapy -May cause drowsiness, dizziness, bradycardia, severe rebound HTN, constipation, nausea, vomiting, urine retention, weight gain, loss of libido -off label use for severe bladder hyperactivity and spinal spasticity

Etidronic Acid (Didronel)

-Biphosphonate -Bone Growth Stimulator -Indicated for paget disease, prevent/treat bone problems after hip replacement surgery -Use caution in patients with renal impairment/liver disease -may cause GI symptoms, headache, allergic reactions

Alendronate (Fosamax)

-Bone growth stimulator -Bisphosphonate/Antiosteoporotic -Indicated for osteoporosis in post menopausal women, increase bone mass in men with osteoporosis, paget disease, prevent osteoporosis -use caution in patients with renal impairment/liver disease -Correct hypocalcemia prior to treatment start -May cause headache, GI Symptoms, and severe musculoskeletal pain

Diazepam (Valium)

-CNS Depressant -Anxiolytic/Benzodiazepine -Controlled Substance IV -Indicated for anxiety, acute alcohol withdrawl, preop sedation, muscle spasm, seizures -May cause drowsiness, bradycardia, headache, incontinence, urine retention, constipation, dry mouth, respiratory depression -Avoid alcohol

Alprazolam (Xanax)

-CNS depressant -Anxiolytic/Benzodiazepines -Indicated for anxiety, panic disorders -May cause dizziness, headache, drowsiness, suicide, impaired coordination, fatigue, palpitations, chest pain, diarrhea, constipation, difficulty urinating, upper respiratory infection

Methylprednisolone

-Corticosteroid/Glucocorticoid -Indicated for severe inflammation or immunosuppression -May cause euphoria, insomnia, headache, heart issues, glaucoma, peptic ulceration, GI irritation, osteoporosis, weight gain

osteoporosis risk factors

-Dietary pattern; Decreased calcium intake is a risk factor for the development of osteoporosis. -Gender; Osteoporosis occurs more often among women than men. -Body mass index; Osteoporosis is more common among individuals who have small body frames. -Hyperthyroidism, as well as excessive intake of medications used to treat hyperthyroidism, increases the risk for developing osteoporosis.

The nurse is collecting data on clients at a clinic. One client has risk factors for substance abuse. What physical sign or signs did the nurse assess that suggest substance abuse in this​ client? Select all that apply.

-Dilated pupils -Odor of alcohol on breath -low body weight

Celecoxib (Celebrex)

-NSAID -Indicated for Osteoarthritis, rheumatoid arthritis, acute pain, and gout -May cause GI Distress, bleeding, allergic reactions, headache, dizziness, dyspnea, monitor for liver toxicity

Naproxen (Aleve)

-NSAID -Indicated for acute gout, acute tendonitis, bursitis, pain, osteoarthritis, rheumatoid arthritis -May cause GI distress, bleeding, and allergic reactions -Monitor labs, especially renal labs -monitor neurological symptoms

ibuprofen (Advil)

-NSAID -Indicated for rheumatoid arthritis, osteoarthritis, mild to moderate pain, fever, migraine -May cause GI Distress, bleeding, and allergic reactions -May interfere with lab tests -Can mask signs and symptoms of infection

Oxycodone/Acetaminophen (Percocet)

-Opioid Analgesic -=Controlled Substance Type II -Indicated for moderate to moderately severe pain, acute pain -May cause paresthesia, dizziness, constipation, nausea, vomiting, edema, anaphylactic, shock, liver injury, respiratory depression, addiction

Morphine (MS Contin)

-Opioid Analgesic -Controlled Substance Type II -Antidote=Narcan -Indicated for moderate to severe pain -Triple check med/dose to prevent overdose -Do not crush extended release -May cause dizziness, drowsiness, euphoria, light headedness, sedation, hallucinations, bradycardia, cardiac arrest, shock, constipation, nausea, vomiting, respiratory arrest/depression, apnea, may cause addiction

hydromorphone (Dilaudid)

-Opioid Analgesic -Controlled Substance Type II -Indicated for management of severe pain -Avoid alcohol -May caused addiction, overdose, GI Distress, sedation, headache, cardiac arrest, urine retention, constipation, respiratory depression

Hydrocodone with Acetaminophen (norco)

-Opioid Analgesic -Controlled Substance Type II -Indicated for moderate to moderately severe pain -May cause GI Distress, constipation, light headedness, dizziness, impairment of mental and physical performance, bradycardia, cardiac arrest, renal toxicity, respiratory depression, allergic reaction

Integumentary affects of immobility

-Pressure Ulcer - damage to underlying tissues -Interface Pressure - tissues compressed between bones and a support surface -Shear - damage from sliding down -Friction Injury - skin rubbed off -Excoriation - abrasion, wound drainage, feces, urine, chemicals -Maceration - contact with moisture

CT scan (computed tomography)

-a series of x-ray photographs taken from different angles and combined by computer into a composite representation of a slice through the body -with or without contrast -detailed images of internal organs -more detailed than x-rays -used to diagnose disease or injury, or plan for med/surgical treatment

Phosphate

2.4-4.4 mg/dL

glucose

70-120 mg/dL

Calcium

8.6-10.2 mg/dL

PaO2

80-100

Chloride

98-106 mEq/L

The nurse is performing a yearly health screening on a patient at risk for osteoporosis. Which clinical assessment finding should the nurse associate with osteoporosis?

A decrease in height over time Rationale:A clinical assessment finding associated with osteoporosis is a decrease in height over time. An increase in weight is not associated with osteoporosis. Chronic episodes of vertebral pain or vertebral pain occurring with substantial movement are not associated with osteoporosis. Osteoporosis is characterized by acute, not chronic, episodes of vertebral pain.

Sedation

A drug that calms a patient, easing agitation, and permitting sleep

Active Transport

A method that requires additional energy (ATP) to move substances against the concentration gradient (low to high)

alcoholism

A primary, chronic disease with genetic, psychosocial, and environmental factors influencing its development and manifestations. The disease is often progressive and fatal. It is characterized by: 1) Impaired control over drinking, 2) Preoccupation with the drug alcohol, 3) Use of alcohol despite adverse consequences, and 4) Distortions in thinking, most notably denial. (Note: Each of these symptoms may be continuous or periodic).

Muscle Relaxation

A relaxation technique that involves consciously tightening and then relaxing each muscle progressively from either head to toe or toe to head.

The nurse is reviewing the admission orders for an older adult client who is being admitted for a hysterectomy. The​ client, who has been diagnosed as having uterine​ cancer, has chronic pain caused by arthritis. The healthcare provider has prescribed​ long-acting oral narcotic medication to be administered every 4 hours. What should the nurse do when providing the medication to the​ client? A. Administer the medication every 4 hours around the clock. B. Consult the provider to order intravenous pain medication. C. Administer the medication sparingly to avoid narcotic addiction. D. Administer the medication if the client requests it.

A. Administer the medication every 4 hours around the clock

The nurse is planning to teach a class regarding factors that influence food choices. Which factor should the nurse​ include?

A. Emotion B.Cost D. Availability E. Convenience

The nurse reviews concepts that are related to comfort with a group of nursing assistants. Which statement should the nurse include in​ teaching? (Select all that​ apply.) A. "Tissue integrity is related to comfort in that decreased tissue integrity increases the risk for​ pain." B. ​"Ethics is related to comfort in that healthcare providers may be reluctant to prescribe opioids based on​ race." C. ​"Inflammation is related to comfort in that inflammation causes​ pain." D. ​"Mobility is related to comfort in that decreased mobility is often caused by​ pain, injury, or​ disease." E. ​"Grief and loss is related to comfort in that loss or expected loss of a loved one creates physical​ discomfort."

A. Tissue integrity is related to comfort in that decreased tissue integrity increases the risk for pain B. Ethics is related to comfort in that healthcare providers may be reluctant to prescribe opioids based on race C. Inflammation is related to comfort in that inflammation causes pain D. Mobility is related to comfort in that decreased mobility is often caused by pain, injury, or disease

A nurse is caring for a client with a sleep disorder. Which question should the nurse ask about the current​ problem? (Select all that​ apply.) A. When did your discomfort start? B. How would you describe your discomfort C. Have you had past experiences that affect the way you view this discomfort? D. Which activities make the discomfort better or worse E. How long have you had this discomfort?

A. When did your discomfort start? B. How would you describe your discomfort? D. Which activities make the discomfort better or worse E. How long have you had this discomfort?

Which of the following statements describes all​ pain? A. Pain's effects can be verbalized. B. ​Pain's effects are primarily​ physiological, not mental or emotional. C. Pain is the result of tissue damage. D. Pain can be localized to a particular area of the body.

A. pain's effects can be verbalized

The public health nurse is providing community education aimed at promoting nutritional habits that decrease an​ individual's modifiable risk factors for heart disease. Which topics should the nurse include in this teaching​ session?

A.Preparing balanced meals B. Selecting lean protein sources C. Strategies for maintaining recommended daily caloric intake D. Benefits of consuming fruits and vegetables

Captopril

ACE inhibitor, antihypertensive -Indicated for HTN, diabetic nephropathy, left ventricular dysfunction after MI, and heart failure -May cause dizziness, headache, tachycardia, hypotension, abdominal pain, constipation, nausea, vomiting, cough, dyspnea, angioedema -Increase urine output

Lisinopril

ACE inhibitor, antihypertensive -Indicated for treatment of HTN -May cause dizziness, headache, hypotension, diarrhea, nausea, dyspnea, angioedema -may cause impaired renal function, impotence, increased urine output -Use caution in patient with renal impairment

Octreotide (Sandostatin)

Antidiarrheal -Growth hormone/Somatostatin -indicated for treatment of flushing and diarrhea from carcinoid tumors, watery diarrhea from vasoactive tumors, acromegaly, dumping syndrome, GI Variceal hemorrhage -may cause nausea, vomiting, loose stools, constipation, gas, bloating, dizziness, headache

Loperamide (Imodium)

Antidiarrheal -Treatment of diarrhea, IBS -may cause dizziness, drowsiness, constipation

Bismuth subsalicylate (Pepto-Bismol)

Antidiarrheal -slows the growth of bacteria that might be causing diarrhea -side effects are rare; most common are darkening of the stools and/or tongue -avoid if allergic to NSAIDs

Lomotil (diphenoxylate/atropine)

Antidiarrheal -treatment of diarrhea -slows the gut -do not use to treat C. Diff -may cause drowsiness, dizziness, headache, dry mouth, loss of appetite -caution in patients with angle closure glaucoma -avoid alcohol

Verapamil

Antihypertensive-Calcium Channel Blocker -Indicated for vasospastic angina, A. Fib, HTN -May cause dizziness, HF, hypotension, constipation, nausea -Decreases smooth muscle contractility in the bladder

Nifedipine (Procardia)

Antihypertensive-Calcium Channel Blocker -Indicated for vasospastic angina, HTN, ureteral calculi -May cause peripheral edema, dizziness, constipation, dyspnea, difficulties in balance -Decreases smooth muscle contractility in the bladder

What nursing assessment question would best indicate that an incontinent man with a history of prostate enlargement might not be emptying his bladder adequately?

Asking the patient "Do you dribble urine constantly?"

A client is transferred from the critical care area to a general medical unit. What action should the nurse take first to help promote​ sleep?

Assess the client's individual circadian rhythm

A patient is prescribed sodium oxybate to treat a sleep disorder. Which instruction should the nurse include when providing medication teaching?

Avoid a diet with a high fat content. Rationale:The nurse will teach the patient to avoid a high-fat diet. A high-fat diet in combination with sodium oxybate decreases the absorption of the medication. The patient taking estazolam or ramelteon should be instructed to avoid grapefruit and grapefruit juice. Zolpidem interacts with all food in general and should not be taken with food or right after a meal.

The nurse is creating a pain management plan using the​ three-step approach for a client with intractable pain. Which interventions should the nurse include in this​ plan? Select all that apply. A. Administer analgesics upon client request. B. Administer a nonopioid with an opioid second. C. Administer a nonopioid analgesic first. D. Administer an opioid analgesic last. E. Administer an opioid analgesic first.

B. Administer a nonopioid with an opioid second. C. Administer a nonopioid analgesic first D. Administer an opioid analgesic last

A client is scheduled for polysomnography test. Which data should the nurse expect to be collected during the test. Select All That Apply: A. Urinary output B. Eye Movements C. Leg movements D. Oxygen Saturation E. Heart rate

B. Eye movements C. Leg movements D. Oxygen saturation E. Heart rate

A patient with osteoporosis has been prescribed calcium citrate supplements. Which topic should the nurse include in the patient's medication teaching?

Best taken with meals Rationale:Calcium citrate supplements may cause indigestion and should be taken with meals. Calcium supplements are not associated with depression, weight gain, or anemia.

Tiludronate (Skelid)

Bisphosphonate -Bone Growth Stimulator

Which assessment finding should the nurse note in a patient diagnosed with urinary incontinence?

Bladder Bulging rationale:During the physical examination for a patient experiencing urinary incontinence, the nurse may find perineal redness, physical or cognitive limitations, and bladder bulging. The use of alternative therapies is assessed during the patient's health history. Bowel sounds are not assessed during a focused urinary assessment. An enlarged prostate tends to cause problems with urinary retention, not incontinence.

Which factor in a patient's history increases the patient's risk for osteoarthritis?

Body mass index (BMI) of 26.4 Rationale: A body mass index (BMI) of 26.4 classifies this patient as overweight, which is a risk factor for OA. Race is not considered a risk factor for OA. A family history of rheumatoid arthritis and diabetes increases the risk for those diseases but not for OA. If the patient had diabetes, the risk would be increased. Smoking increases the risk for gout but not for OA.

Stool/Feces

Body wastes and undigested food eliminated from the bowle

The nurse is prioritizing care for a patient with a sleep-rest disorder. Which nursing diagnosis should the nurse identify as being the highest priority?

Breathing Pattern, Ineffective Rationale:An altered breathing pattern can be a safety risk. Patient care should prioritize considerations that pose actual or potential risks to the patient's safety. (NANDA-I © 2014)

What is the relationship between a full opioid agonist and the ceiling​ effect? A. A client may use a full opioid agonist as much or as little as necessary to control chronic pain with no ill effects. B. At some​ point, a full opioid​ agonist's side effects cease to increase in​ potency, but the​ pain-relieving effect continues to increase. C. Side effects may limit a full opioid​ agonist's use but not a plateau in the beneficial effects it produces. D. A full opioid agonist produces few withdrawal symptoms when the​ drug's effects plateau and the client begins easing off the drug.

C. Side effects may limit a full opioid​ agonist's use but not a plateau in the beneficial effects it produces.

Lorazepam (Ativan)

CNS Depressant -Controlled Substance Type IV -Anxiolytic/Benzodiazepine -Indicated for treatment of anxiety, insomnia, pre-op sedation, status elliptus -may cause drowsiness, dizziness, hypotension, abdominal discomfort, nausea, respiratory failure

The nurse is caring for the patient with narcolepsy. Which category of medication should the nurse expect to be prescribed for this patient?

Central nervous system (CNS) stimulant Rationale:A CNS stimulant may be prescribed for patients with narcolepsy or obstructive sleep apnea. Medications for patients with restless leg syndrome include an anti-Parkinson agent and an anticonvulsant. Anxiolytics may be prescribed as a short-term treatment for patients who experience insomnia.

The nurse is planning care for a client with osteoarthritis. What nursing diagnosis would have the highest priority?

Chronic pain

A 60-year-old patient presents with multiple episodes of fecal incontinence over the past week. Which condition should the nurse consider related to decreased muscle tone and rectal sensation?

Constipation Rationale:Attributes related to structural integrity of the intestinal tract can include decreasing muscle tone and rectal sensation from cumulative local trauma resulting from childbirth, constipation, rectal impaction, or other conditions that can affect the structures of the intestinal tract. Older adults are at increased risk for fecal incontinence due to chronic disease, polypharmacy, and fecal impaction from inactivity or immobility and reduced fluid intake.

A client with osteoarthritis tells the nurse she has difficulty walking to the bathroom first thing in the morning. What nursing action would assist this client?

Consulting with physical therapy for an assistive walking device such as a walker or a cane

To assess whether there is any improvement in a patient's dysuria, which question will the nurse ask? a. "Do you have to urinate at night?" b. "Do you have blood in your urine?" c. "Do you have to urinate frequently?" d. "Do you have pain when you urinate?"

D. Do you have pain when you urinate?

After several days of antibiotic therapy, an older hospitalized patient develops watery diarrhea. Which action should the nurse take first? a. Notify the health care provider. b. Obtain a stool specimen for analysis. c. Teach the patient about handwashing. d. Place the patient on contact precautions.

D. Place the patient on contact precautions

The nurse should identify which risk factor as contributing to bowel incontinence?

Damage to a nerve Rationale:Individuals with nerve damage, including multiple sclerosis and spinal cord injury, are at risk of bowel incontinence. Such recommended dietary measures as consuming a high-fiber diet and ample fluids to maintain a soft-formed stool, or maintaining a low-residue diet to reduce the number of stools, may be beneficial. Kegel exercises to improve sphincter and pelvic floor muscle tone may be of long-term benefit.

Pseudophedrine (Sudafed)

Decongestant/Alpha agonist -off label use: treatment of stress incontinence - activates alpha receptors in bladder neck and proximal urethra to increase smooth muscle tone

The nurse is teaching a group of older adults with osteoarthritis about dietary interventions that may reduce disease progression. Which recommendation should the nurse include?

Decreasing caloric intake while increasing caloric expenditure Rationale:The nurse should promote balanced nutrition for patients with osteoarthritis (OA) since being overweight and obese can add to disease progression. The nurse should instruct patients to decrease caloric intake and increase caloric expenditure. There is no strong evidence to support the use of supplements, so the nurse should not include that in the teaching session. Increasing protein-rich foods or eating several smaller meals a day will not impact the progression of OA.

An elderly patient comes to the hospital with a complain of severe weakness and diarrhea for several days. What is the most important problem to assess first?

Dehydration

Which of the following terms refers to a physiologic need for a substance that the client cannot control and that results in withdrawal symptoms if the substance is​ withheld?

Dependence

The nurse is caring for a patient diagnosed with urinary retention. Which medication on the patient's medical administration record should the nurse question?

Diphenhydramine hydrochloride Rationale:The nurse would question the use of an antihistamine, such as diphenhydramine hydrochloride, for a patient with urinary retention. These medications can affect the autonomic nervous system and interfere with the normal urination process. Bethanechol chloride is a medication used to treat urinary retention. Acetaminophen and ibuprofen can be administered safely for a patient with urinary retention.

An older adult client is brought into the emergency room after experiencing a fall. The nurse suspects elder abuse. Which assessment findings support the​ nurse's suspicions?

Dislocations Poor hygiene Dehydration Fecal impaction

The nurse caring for a patient with osteoporosis notes that the patient's height has significantly decreased each year over the past few years. Which further additional assessment finding should the nurse anticipate?

Dorsal kyphosis Rationale:Along with loss of height, characteristic dorsal kyphosis and cervical lordosis develop, accounting for the buffalo hump (sometimes referred to as a "dowager's hump") frequently associated with aging. Recumbent is a position, and cervical kyphosis and thoracic lordosis do not occur with osteoporosis.

The nurse is providing medication teaching to a patient who has been prescribed alendronate sodium (Fosamax) for the treatment of osteoporosis. Which adverse effect should the nurse include?

Dyspepsia Rationale:Dyspepsia is a common adverse effect that occurs for patients taking alendronate sodium (Fosamax). Sinusitis and hot flashes are expected adverse effects that occur with raloxifene hydrochloride (Evista). Diarrhea, not constipation, is an adverse effect expected with the medication.

The nurse is caring for a bedbound female patient. Which intervention should the nurse implement to support voiding and avoid urine retention in the female patient?

Elevating the head of the bed Rationale:Elevating the head of the bed allows the patient to sit in a more natural position. A urinal would be provided to the male patient. Providing privacy for the patient and warming the bedpan help to promote voiding.

The nurse plans care for a patient with restless leg syndrome. Which intervention should the nurse identify as being the most effective for this health problem?

Encourage a routine exercise program. Rationale:Clinical therapies for a patient with restless leg syndrome include pharmacologic therapy, gentle stretching, massage, exercise therapy, and good sleep hygiene. Counseling the patient on nutritional intake and weight reduction is an effective clinical therapy for the patient with obstructive sleep apnea. Sleep-restriction therapy and relaxation therapy are effective clinical therapies that can be utilized for the patient experiencing insomnia.

The nurse is providing teaching to a patient diagnosed with osteoporosis about how to slow the disease process. Which information is the most appropriate to provide?

Encouraging smoking cessation Rationale:The information that is the most appropriate for the patient with osteoporosis to decrease the progression of the disease is to encourage smoking cessation. Smoking decreases the blood supply to bones, and nicotine slows the production of osteoblasts and impairs the absorption of calcium, contributing to decreased bone density. Decreasing fluid intake, encouraging the regular use of pain medication, and discouraging further physical activity will not decrease the progression of the disease.

The nurse is assessing a patient who is postmenopausal and at risk for osteoporosis. The nurse notes a 3-in decrease in height from last year's assessment. Laboratory and radiological studies confirm osteoporosis. Which collaborative intervention should the nurse anticipate to further prevent decrease in bone loss?

Estrogen replacement therapy Rationale:Pharmacotherapy is used for prevention and treatment of osteoporosis. Estrogen replacement therapy reduces bone loss, increases bone density in the spine and hip, and reduces the risk of fractures in postmenopausal women. Vitamin D and calcium supplementation, as well as bisphosphonates are beneficial to any patient with osteoporosis, but postmenopausal women can be specifically treated with estrogen replacement therapy.

An older adult client with bilateral osteoarthritis of the knees tells the nurse "I know I need to lose weight, but exercising makes my knees ache". What instruction should the nurse provide to this client?

Exercising the muscles in your legs might be hard now, but over time, it will help protect your knees

Excessive daytime sleepiness (EDS)

Extreme fatigue felt during the day. Signs of this include falling asleep at inappropriate times such as while eating, talking, or driving. May indicate a sleep disorder.

The nurse assessing a patient who is diagnosed with osteoarthritis of the hands finds bony lumps on the end joints of the digits. How should the nurse document this finding in the medical record?

Heberden nodes Rationale:Heberden nodes are bony lumps occurring at the end joint of the digits in a patient with osteoarthritis (OA). Bouchard nodes occur in the middle joint of the digits. Osteoblasts are cells that form new bone. Lymph node swelling does not occur in the hands.

The nurse is assessing a patient with osteoarthritis. Which is a priority assessment that the nurse should include in relation to the diagnosis?

Heberden or Bouchard nodes Rationale:Patients with osteoarthritis often have Heberden or Bouchard nodes that the nurse should assess for. Assessment of sensory function, vascular function, and for trophic changes would not be indicated by this diagnosis.

A 62-year-old female patient presents with a possible wrist fracture and reports no other health problems. This is the third such injury that the patient has had in the past year, and the nurse suspects osteoporosis. Which assessment finding in the patient's health history supports the diagnosis of osteoporosis?

Hx of Alcoholism Rationale:A history of alcoholism places this patient at risk for osteoporosis. An inactive lifestyle, not an active lifestyle, would place this patient at risk. Being underweight, not overweight, is a risk factor. A lack of vitamin D intake, not daily intake, is another risk factor for osteoporosis.

The nurse notes that a patient has a history of narcolepsy. How should the nurse classify this sleep-rest alteration?

Hypersomnia Rationale:Narcolepsy is a severe form of hypersomnia. This classification of sleep-rest disorder is characterized by extreme daytime drowsiness despite getting adequate sleep. Insomnia is when an individual has trouble falling asleep or staying asleep. Dyssomnias include restless leg syndrome and periodic limb movement disorder. Parasomnias are unpleasant or undesirable behaviors that occur anytime during sleep.

The nurse is evaluation care provided to a client with osteoarthritis. What statement by the client indicates to the nurse that interventions for OA has been successful?

I am sleeping throughout the night and have not missed any work because of knee pain

The nurse is obtaining a health history on a patient diagnosed with osteoporosis. Which patient statement has the strongest association with osteoporosis?

I try to walk twice a week. Rationale:The statement made by the patient that has the strongest association with osteoporosis is, "I try to walk twice a week." Walking is a weight-bearing exercise, but it is recommended that the patient will participate in weight-bearing exercises for approximately 30 minutes a day at least four days a week. The statements regarding lactose intolerance, abstinence from smoking, occasional alcohol use, and consuming a vegan diet are not risk factors for osteoporosis.

A 65-year-old man with a low testosterone and lifetime calcium level has had two bone fractures in the past 2 years. Which intervention should the nurse suggest to prevent or slow the development of osteoporosis?

Increasing calcium intake Rationale:The nurse will recommend that the patient increase calcium intake as a way to slow the development of osteoporosis. The use of corticosteroids increases the risk factor for osteoporosis. Low estrogen levels are a cause of osteoporosis in women but not in men, so estrogen supplements would not be indicated for an older man. The patient should be advised to increase activity, not to decrease it.

A client with chronic hip pain is diagnosed with osteoarthritis. What instruction regarding home safety is most appropriate for the nurse to provide this client?

Install grab bars in the bathroom near the commode and in the shower

An adult client diagnosed with sleep apnea has been prescribed a continuous positive airway pressure​ (CPAP) machine as treatment. The nurse is instructing the client on how to use the machine. Which instruction least relates to ensuring the​ patient's comfort with the​ device?

Instruct the client to wear the mask with air pressure while sleeping.

An older adult female patient is diagnosed with osteoporosis. Which risk factor should the nurse recognize as a contributing to this disease?

Lack of vitamin D Rationale:A patient with a history of decreased levels of vitamin D will be at a risk of developing osteoporosis. This is a modifiable risk factor for osteoporosis. Low testosterone in male patients, not female patients, increases the risk of osteoporosis. A diet rich in calcium and weight-bearing exercise both decrease the risk of osteoporosis.

Citrucel

Laxative (bulk-forming) -indicated for treatment of constipation -may cause gas, abdominal cramping

Colace (docusate)

Laxative/Emollient -indicated for treatment of occasional constipation -1st method used for prevention and treatment of constipation -used when straining should be avoided -may cause abdominal pain, diarrhea, cramping, irritated throat

Emollient

Laxatives that contains surfactant that helps to "wet" and soften the stool

Abandonment

Leaving a patient after care has been initiated and before the patient has been transferred to someone with equal or greater medical training.

Factors affecting mobility

Lifestyle Factors, Environmental Factors, Developmental Factors, Physiological Factors

Suicide Screening

Look for: depressed mood (or sudden elevated mood after being depressed), unusually fast or slow movements, patient expressions of suicide or finality ("I won't be around to deal with that."), unusual expression of anger Ask about: changes in weight, sleep, or concentration Suspicious behavior: getting affairs in order, giving away personal belongings, obtaining a weapon, increased alcohol or drug use, telling a friend, family or healthcare professional about contemplating suicide, sudden violent behaviors What to do: practice therapeutic interviewing to gather more information about circumstances, suicide plan, and actions already taken; assess patient's ability to keep a meaningful contract; do not leave patient alone, involve the family if patient consents (contact family if under 16), admit for psych eval Tools: Beck Depression Inventory, Geriatric Depression Scale, Hopelessness Scale, Suicide Attempt Self-Injury Review, Suicidal Ideation Questionnaire (SIQ) for ages 13-18 or the Columbia-suicide Severity Rating Scale (C-SSRS)

The nurse is teaching a postmenopausal woman how to reduce the risk factors associated with osteoporosis. Based on the patient's history, which activity should the patient be advised?

Maintaining calcium intake of 1000-1500 mg per day Rationale:The postmenopausal patient should be encouraged to maintain a calcium intake of 1000-1500 mg daily, through either diet or a calcium supplement. The National Institutes of Health recommends a daily vitamin D intake of 800-1000 IU for those aged 50 and older.

A patient taking medication for insomnia reports drinking several cups of coffee each day, exercising only once per week or less, and leaving the television on throughout the night. When providing the discharge teaching, which information should the nurse emphasize to this patient?

Medication side effects Rationale: Nursing must implement interventions related to potential safety risks. Exercise habits, environmental distractions, and daily caffeine consumption are appropriate topics for inclusion in the patient's discharge teaching. However, side effects of sleep aids often include residual drowsiness and somnolence, both of which can increase the patient's risk for injury. As such, it is essential to teach the patient about the side effects of the medication.

A patient asks the nurse about the use of heat and cold therapies to address the swelling associated with osteoarthritis. Which therapy should the nurse encourage the patient to use?

Mild Cold Rationale:For swelling, the nurse would tell the patient to use mild cold. Deep cold would be used to address pain. Heat would be used to address pain and improve flexibility.

The nurse is providing dietary teaching for a patient newly diagnosed with osteoporosis. Included in the teaching is the importance of dietary intake of calcium and vitamin D. Which foods that are high in vitamin D should the nurse recommend?

Milk Rationale:Vitamin D is necessary for the body to absorb calcium. The food the nurse will recommend that is high in vitamin D is milk. Milk is also high in calcium, which is recommended in the prevention of further complications of osteoporosis. Beef does not contain high amounts of vitamin D. Orange juice and beans contain high amounts of calcium.

The nurse is caring for a patient diagnosed with osteoporosis. When planning the patient's care, which nursing diagnosis is most appropriate?

Mobility: Physical, Impaired Rationale:A nursing diagnosis that is most appropriate for a patient with osteoporosis is Mobility: Physical, Impaired. The patient may experience impaired mobility due to fractures and acute pain. The other diagnoses are not appropriate for a patient with osteoporosis. (NANDA-I © 2014)

The nurse is teaching a group of staff members about sleep-rest disorders. Which example should the nurse use when explaining hypersomnia?

Nacrolepsy Rationale:Narcolepsy is classified as hypersomnia and is characterized by extreme daytime drowsiness despite getting sufficient sleep. Dyssomnia is also known as restless leg syndrome. Enuresis, bedwetting, is classified as a parasomnia that is an abnormal action during sleep. Insomnia is the inability to fall or stay asleep.

The nurse is teaching a group of community members about risk factors for developing sleep-rest disorders. Which risk factor for obstructive sleep apnea should the nurse emphasize?

Obesity Rationale:Obesity is the primary risk factor for the development of obstructive sleep apnea. Female gender, age 60 years or older, and mental health disorders are risk factors for the development of insomnia.

An older adult patient with a history of osteoarthritis (OA) reports that the acetaminophen therapy is not working anymore. Which should the nurse do first?

Obtain a list of current medications and supplements being used. Rationale:Older adults with osteoarthritis (OA) should first have a complete medication history including the use of supplements because of the risk of polypharmacy. Nonsteroidal anti-inflammatory drugs (NSAIDs) are not considered safe for older adults and should not be recommended. Narcotics are the second-line medication for OA, but asking the provider to prescribe a narcotic would not be the first action. The nurse should not encourage the patient to use supplements; there is little scientific evidence that supports use of supplements for disorders such as this.

The nurse is providing teaching to the client recently diagnosed with osteoarthritis. What statement by the nurse is correct?

Osteoarthritis occurs due to erosion of cartilage in the joints

Osteoporosis and increase risk of bone fractures

Osteoporosis causes a loss of bone matrix. -a loss of bone matrix and a decrease in bone mineralization characterizes osteoporosis.

The healthcare provider is discussing a surgical intervention to realign a hip joint for a patient with osteoarthritis. Which surgical treatment should the nurse anticipate will be recommended?

Osteotomy Rationale:An osteotomy is an incision into or through the bone that can realign or shift the joint load into a better aligned point or toward a less damaged part of the joint. A joint arthroplasty reconstructs or replaces the affected joint when it is severely damaged and pain seriously alters lifestyle. An arthroscopy is used to debride the rough and irregular damaged cartilage. Although fluid may be aspirated from a joint for analysis, joint cartilage is not biopsied to realign a joint affected by osteoarthritis.

A patient with osteoarthritis reports increasing knee pain. Which medication should the nurse anticipate will be prescribed by the healthcare provider?

Prescription-strength nonsteroidal anti-inflammatory drug (NSAID), such as celecoxib Rationale:Celecoxib (Celebrex) is a prescription nonsteroidal anti-inflammatory drug (NSAID), COX-2 inhibitor that relieves pain associated with osteoarthritis. Naproxen (Aleve) does relieve pain and stiffness associated with osteoarthritis; however, it is an over-the-counter NSAID, which may not have provided relief for the patient. Acetaminophen is a first-line therapy for pain; however, it is an over-the-counter medicine and may not provide relief for the patient. Morphine sulfate is used for patients experiencing moderate to severe pain but should not be used before trying celecoxib.

A parent reports that their 5-year-old child has never been able to achieve bowel control. Which condition should the nurse suspect?

Primary encopresis Rationale:Encopresis is an abnormal elimination pattern characterized by recurrent soiling or passage of stool at inappropriate times by a child who should have achieved bowel continence. Children with primary encopresis have never achieved bowel control. Children with secondary encopresis will have bowel continence for several months. Psychosocial issues for children related to bowel complications would be problems with using toilets that are not in their home, being afraid to ask to use the restroom in school, or being so busy that they forget to go to the bathroom. Growth retardation is used to define the statistics on how a child grows and progresses through their adolescence. This particular factor has no effect on bowel incontinence.

The nurse notes that an older adult patient is having difficulty with sleep. Which intervention should the nurse implement to help this patient?

Provide extra blankets. Rationale:Physiologic changes that may contribute to sleep disorders in older adults are changes in circulation, metabolism, and body tissue density. These changes limit the older adult's ability to generate heat and maintain a comfortable body temperature. Nursing interventions used to promote warmth and sleep for older adults include warming the bed with an electric or prewarmed blanket, using flannel sheets instead of cotton or polyester, encouraging the patient to wear warm clothing to bed, and providing extra blankets. Limiting late-afternoon activity can contribute to an older adult losing physical strength; strength loss further disrupts the sleep-wake cycle. It is not necessary to limit nurse-patient interaction in the evening. Limited interaction contributes to sleep disruption. Soft music at bedtime contributes to nighttime noise and sleep disruption.

The nurse is planning care for a client with osteoarthritis of the hip. What intervention would be appropriate for this client?

Provide moist heat packs to the affected joint 3 times each day

A 74 year old male patient tells the nurse that growing old causes constipation so he has been using a suppository for constipation every morning. What action should the nurse take first?

Question the patient about risk factors for constipation

The nurse is assessing the mobility of a patient newly diagnosed with osteoarthritis. Which assessment should the nurse conduct that will be used to most accurately determine disease progression over time?

Range of motion of affected joints Rationale:Assessment of the range of motion (ROM) of affected joints at baseline is important for future comparison. ROM should be assessed at each subsequent nurse-patient interaction. Assessment of current functional abilities, pain, and impact of the osteoarthritis (OA) on sleep and rest are also important but will not provide the most accurate information to determine disease progression as the nurse continues to work with the patient.

A patient with a long history of osteoarthritis reports increasing pain in the shoulder joints. Which assessment should the nurse perform to determine if the patient needs diagnostic testing?

Range of motion of the shoulder joints compared with the baseline Rationale:When a patient reports increased pain in the affected joint during evaluation, the nurse should also evaluate range of motion (ROM) and compare it with baseline ROM. If pain has increased and ROM is decreased, an x-ray may be indicated to monitor joint space and osteophyte formation. Evaluation of ability to complete activities of daily living (ADLs) and impact of the osteoarthritis (OA) on the patient's lifestyle and on sleep and rest are also important but would not provide information concerning the need to obtain an x-ray.

A nurse is caring for a patient with functional incontinence. What would be an appropriate intervention to implement?

Reduce environmental barriers

A patient tells the nurse, "My osteoarthritis is beginning to interfere with my ability to work despite adhering to my current treatment plan." Which collaborative or independent action should the nurse perform?

Refer the patient to occupational therapy Rationale:Treatment of osteoarthritis (OA) requires interprofessional care from nurses, primary care providers, rheumatologists, and physical and occupational therapists, among others. Since the patient is just beginning to complain of interference, the best approach would be a consultation from an occupational therapist to help relieve pain and maintain function. An MRI and/or surgical intervention would not necessarily be indicated at this time based on the patient's complaint. Increasing the use of over-the-counter analgesics would not necessarily be appropriate, nor would it directly address the limitations occurring at work.

Micturtion/Voiding/Urination

Releasing urine from the urinary bladder

A patient is admitted to a clinic with urinary retention caused by a mechanical obstruction. The nurse should suspect which condition as the likely cause of the patient's condition?

Repeated UTIs Rationale:Repeated urinary tract infections lead to scarring of structure, which is a functional problem associated with urinary retention. Either mechanical obstruction of the bladder outlet or a functional problem can cause urinary retention. Scarring caused by repeated UTIs subsequently leads to urethral stricture and produces mechanical obstruction. Benign prostatic hypertrophy and fecal impaction are the causes of an obstruction that will lead to urinary retention. Anticholinergic medications may cause retention, but this is not a functional problem. Once the medication is stopped, the urinary retention resolves.

A patient asks the nurse about risk factors for the development of osteoarthritis. Which factor should the nurse include?

Repeated use of weight-bearing joints Rationale: A major cause of osteoarthritis (OA) is repeated use of weight-bearing joints. Rheumatoid arthritis is a chronic inflammation of peripheral joints. Psoriasis is associated with psoriatic arthritis, but not osteoarthritis. Ingestion of purine-rich substances is contraindicated in gouty arthritis, not osteoarthritis.

A patient with osteoarthritis asks the nurse about lifestyle alterations that avoid overstressing of the joints. Which activity should the nurse suggest?

Resting for short periods throughout the day with the joint in correct alignment Rationale:To decrease overstressing of joints, the patient should rest for short periods during the day with the joint in correct alignment. The patient should perform active range-of-motion exercises daily with all joints; this should not be limited. Acetaminophen should be taken as needed; it will not necessarily reduce the possibility of overstressing a joint. Cold packs reduce pain and swelling; they are not indicated for use before activity to avoid overstressing joints.

A patient with narcolepsy seeks medical attention for the disorder. Which teaching material should the nurse prepare that focuses on the most effective treatment for narcolepsy?

Sleep Hygiene Rationale:Common therapy to treat narcolepsy includes good sleep hygiene and counseling. Losing weight, avoiding alcohol, and surgery are treatment options for obstructive sleep apnea.

A patient requests information to improve sleep. Which should the nurse include when teaching this patient?

Sleep Hygiene Rationale:Good sleep hygiene is the main preventive measure for sleep-rest disorders. Exercise and pharmacologic therapy are recommended for patients with insomnia and restless leg syndrome. Cognitive-behavioral therapy is recommended for patients with insomnia.

A pregnant client is having difficulty sleeping because of the weight of the baby and the need to void frequently throughout the night. Which instruction should the nurse include to help this patient with sleep?

Sleep Hygiene Practices Rationale:Because many medications can harm the developing fetus, the best treatment for pregnant women with sleep disorders is practicing good sleep hygiene. The use of valerian is contraindicated in pregnancy and breastfeeding. Restricting fluid intake can result in dehydration and preterm labor. Nursing interventions that promote warmth are used for older adults. The obstetrical patient may be uncomfortable with additional heat due to the increase in hormones and metabolism that occur during pregnancy.

The nurse prepares to assess a patient who is experiencing symptoms of a sleep-rest disorder. Which data should the nurse collect as part of the patient's sleep history?

Sleeping position Rationale:The sleep history includes asking the patient about positioning while sleeping. Although drug regimen is included in the patient's health history, this component is not part of the sleep history. Vital signs and a neurological exam are included as part of the patient's physical assessment.

Assistive Devices

Special equipment that helps a person who is ill or disabled to perform ADLS; also called adaptive devices. -crutches, walkers, canes

castor oil

Stimulant laxative-- reduces absorption of electrolytes and water from gut Indicated for: Constipation; labor induction -May cause abdominal pain, cramping, nausea, diarrhea, weakness

Which diagnostic test will help differentiate osteoarthritis as the cause of joint damage versus other possibilities?

Synovial fluid analysis RationaleAnalyzing the synovial fluid of the involved joint helps to determine the cause of joint damage, which may include inflammatory arthritis and gout. This distinction will determine the factors of treatment. Electromyography analyzes the strength of muscle contraction. It does not provide any information about joint mobility. An MRI of the affected joint determines the extent of joint damage but does not differentiate causes of the damage. An x-ray of the joint determines the presence of osteoarthritis and the potential presence of joint complications.

The nurse is caring for an older adult at risk for osteoporosis who frequently experiences a loss of balance. Which activity should the nurse encourage the patient to do to prevent osteoporosis?

Tai Chi Rationale:The activity that the nurse can encourage the patient to do to prevent osteoporosis is tai chi. Tai chi is beneficial to the patient who has problems with balance. Increasing calcium supplementation will not provide the weight-bearing exercise necessary to increase the osteoblast growth and activity necessary to maintain strong bones. Having the patient walk when they "feel" stable may not provide consistent exercise that is necessary to maintain the health of the bones. An assistive device should be secured for the safety of a patient who is unstable.

The nurse is planning care that will help maintain physical mobility for a patient with osteoarthritis of the hips. Which intervention should the nurse include?

Teaching active and passive range-of-motion exercises Rationale:Teaching active and passive range-of-motion exercises will assist in addressing the potential problem of physical mobility with a patient with osteoarthritis (OA). Encouraging the patient to avoid all physical activity does not address the potential problem of physical mobility for a patient with OA. The patient should be encouraged to exercise to develop supportive muscles and tendons. Recommending a local support group for the patient and spouse and educating the patient about proper skin care do not address the potential problem of limited physical mobility for a patient with OA.

The nurse is teaching a patient diagnosed with osteoarthritis how to prevent severe pain. Which intervention should the nurse include?

Teaching the patient to take mild analgesics on a routine basis Rationale:Teaching the patient to take mild analgesics on a regular basis can best help the patient to manage pain before it becomes too severe. The perception of pain varies from one patient to another, so it would not be appropriate to encourage the patient to use stronger analgesics if they are not needed; this would not be the best approach to prevent severe pain from occurring. Range of motion (ROM) exercises and home safety improvements can both be helpful to manage osteoarthritis (OA) but would not be the best approaches to preventing severe pain from occurring.

A patient diagnosed with osteoarthritis asks the nurse why there is so much joint pain. Which response by the nurse is accurate?

The bones are rubbing against each other and irritating the synovial tissue." Rationale:As an individual ages, cartilage begins to break down, becoming rough and eventually wearing away, which allows the bones to rub against each other. Particles breaking off the joint irritate the synovial tissue, leading to pain, stiffness, inflammation, and swelling. Thus, stating that the pain is due to the particles breaking off is the best answer to this question. The other selections do not describe the pathophysiologic process underlying the pain caused by osteoarthritis. It is not due to issues of instability of the ligaments, overstretching of the muscles, or inflammation of the connective tissue.

The nurse is assessing the nutritional status of an older client. Which finding is most likely to suggest​ xerostomia?

The client has a chronically dry mouth despite adequate intake of fluids.

The nurse plans care for a client with a sleep disturbance. Which should the nurse identify as an appropriate goal for this​ client?

The client will initiate relaxation techniques 45 minutes prior to bedtime.

Alkalosis

The condition that results when hydrogen ion concentration falls below normal and the pH level rises above 7.45

Acidosis

The condition that results when hydrogen ion concentration increases above normal, causing the pH to drop below 7.35

The nurse working in a clinic is screening female adolescent patients for risk factors of osteoporosis. Which patient has the greatest risk for osteoporosis?

The female adolescent on a cross-country running team Rationale:The female that is on the cross-country running team has the greatest risk factor for osteoporosis. Adolescent athletes who participate in sports that emphasize leanness, such as gymnastics or cross-country running, are at risk for osteoporosis. Golf does not pose a great risk factor. A sedentary lifestyle over a long period of time or sedentary activity such as chess increases the risk for osteoporosis in adults.

Elder Abuse

The intentional physical, emotional, or sexual mistreatment or neglect of an individual 65 years or older

osmosis

The movement of water across a selectively permeable membrane from an area of higher concentration to an area of lower concentration.

Urinary Frequency

The need to urinate often, specifically more than 4-6 times a day

Diarrhea

The passage of liquid feces and an increased frequency of defecation

What information obtained by the nurse interviewing a patient is most important to communicate to the HCP?

The patient has noticed blood in the stools

The nurse is providing care for a patient diagnosed with osteoporosis who is recovering from a wrist fracture. Which outcome should the nurse expect the patient to meet?

The patient identifies and eliminates safety hazards. Rationale:A patient with a wrist fracture should be able to identify and eliminate safety hazards to prevent further injury. Achieving adequate calcium and vitamin D intake, incorporating weight-bearing exercises, and maintaining a healthy weight are not expected outcomes at this time.

During a home visit, the nurse is concerned that a patient recovering from an osteoporosis-related fracture is at risk for future fractures. Which assessment finding supports the nurse's conclusion?

The patient is smoking cigarettes. Rationale:For osteoporosis, the nurse can help the patient identify risk factors and encourage actions such as increasing activity, smoking cessation, healthy eating, and taking the recommended amount of calcium and vitamin D each day. Smoking cigarettes increases the patient's risk for a future osteoporosis-related fracture. Smoking decreases the blood supply to bones, and nicotine slows the production of osteoblasts and impairs the absorption of calcium, contributing to decreased bone density. Although the patient should be discouraged from using alcohol, drinking an occasional glass of wine does not support that the patient is at an increased risk for future fractures. Using a treadmill for exercise and consuming fresh produce help reduce the risk of future osteoporosis-related fractures.

The nurse reviews information received during handoff communication from the previous shift. Which patient should the nurse identify as being most at risk for restless leg syndrome?

The patient who has hypoestrogenemia Rationale:The patient who is most at risk for restless leg syndrome is the one who has hypoestrogenemia. Restless leg syndrome (RLS) is a neurologic sensorimotor disorder that is characterized by an overwhelming urge to move the legs when at rest. There is some suggestion that menopausal women are more prone to RLS. Researchers examined the relationship between RLS in postmenopausal women and onset of postmenopausal symptoms. Their findings indicate that the hypoestrogenemia associated with menopause may serve as a risk factor for RLS symptoms. A patient with a hypocretin deficiency is at risk for narcolepsy. Patients with RLS experience unpleasant sensations in their legs. Patients with RLS can control movement in their legs; they just experience an overwhelming urge to move their legs.

Diuresis/Polyuria

The production and excretion of abnormally large amounts of urine

Lab results are back for a client who has limiting joint pain. Synovial fluid analysis shows no uric acid crystals or bacteria. The client asks what the test results mean. How should the nurse respond?

These test results mean that your joint pain is likely not caused by gout or septic arthritis

The nurse is providing patient teaching about the proper use of a continuous positive airway pressure (CPAP) device. Which information should the nurse emphasize in this teaching?

Tighten the face straps so that the mask fits you snugly." Rationale:Patient teaching for using a CPAP or bilevel positive airway pressure (BiPAP) machine includes instructing the patient to ensure proper fitting of the mask to the face. The patient needs to wear the correct size mask and keep the facial straps tightened so hat the mask fits snugly on the face. A humidifier and nasal sprays are recommended for use with CPAP to help prevent drying of the oral and nasal cavities. Patients should be encouraged to become accustomed to the mask by wearing it while awake and when the machine is not in use.

The nurse is caring for a patient with urinary incontinence who has been prescribed bladder-training behavior modification. Which goal of therapy should the nurse include in the teaching session with the patient?

To gradually increase the bladder capacity by increasing the intervals between voiding and resisting the urge to void between scheduled times Rationale:Bladder training increases the bladder capacity by increasing the intervals between voiding and resisting the urge to void between scheduled times. Habit training is toileting on a schedule that corresponds with the normal pattern. Scheduled voiding is toileting at regular intervals. Kegel exercise is a technique that is done to strengthen the pelvic floor muscles.

NREM Sleep happens before REM sleep; True or False

True

No cure for Fibromyalgia; True or False

True

The nurse is caring for a postmenopausal patient who reports difficulty sleeping and low back pain. Which testing procedure should the nurse anticipate being ordered to screen the patient for osteoporosis?

Ultrasound Rationale:The nurse can expect an ultrasound to be prescribed for the screening of osteoporosis. An ultrasound transmits painless sound waves through the heel of the foot to measure bone density and is accurate for screening purposes only. Alkaline phosphatase is elevated following a fracture. A dual-energy x-ray absorptiometry (DEXA) is used for diagnostic purposes and is not a screening tool for osteoporosis. Gla protein is used as a marker for osteoclastic activity and is an indicator of bone turnover. This test is most useful to evaluate the effects of treatment, rather than to indicate the severity of the disease.

A patient reports feeling as if their bladder is always full, requiring frequent trips to the bathroom. The patient also reports difficulty starting their urine stream and a weak urine flow. After a postvoiding catheterization obtained 250 mL of urine, the patient was diagnosed with chronic urinary retention. Which clinical therapy should the nurse anticipate being ordered?

Urethral Dilation Rationale:Urethral dilation is often performed to aid in complete emptying of the bladder. Radiation therapy is used to treat cancers of the urinary system. Lithotripsy is used to reduce kidney stones. Vaginal devices are used to prevent the involuntary leakage of urine.

The nurse notes in the medical record that the patient's incontinence is related to an overactive detrusor muscle. Which type of urinary incontinence should the nurse suspect the patient is experiencing?

Urge Rationale: Urge incontinence is related to an overactive detrusor muscle, which increases bladder pressure. Stress incontinence is related to pelvic muscle relaxation and a weak urethra and surrounding tissues, which cause decreased urethral resistance. Overflow incontinence is related to a lack of normal detrusor muscle function, which causes the bladder to overfill and increases bladder pressure. Functional incontinence is related to the inability to respond to the need to urinate.

Solifenacin

Urinary Antispasmodic -Indicated for overactive bladder with symptoms of urgency, frequency, and leakage -Contraindicated in patients with urinary retention, narrow angle glaucoma -May cause angioedema, anaphylaxis, urinary retention, allergic reaction -Most common side effects: dry mouth, constipation, UTI, blurred vision

Tolterodine

Urinary Antispasmodic Indicated for overactive bladder in patients with symptoms of urinary frequency, urgency, or urge incontinence -May cause headache, fatigue, dizziness, drowsiness, chest pain, dry mouth, abdominal pain, constipation, dysuria, urine retention, weight gain, bronchitis -Assess baseline bladder function, monitor for residual urine, monitor labs

Oxybutynin (Ditropan)**

Urinary Antispasmodic/Antimuscarinics -Indicated for bladder instability associated with voiding, overactive bladder, detrusor over-reactivity -May cause dizziness, hallucinations, headache, palpitations, tachycardia, dry eyes, constipation, nausea, vomiting, urinary hesitancy, urinary retention -Monitor patient for residual urine after voiding -No operating heavy machinery, avoid alcohol, may cause dry mouth

Bethanechol (Urecholine)

Urinary stimulant/cholinergic agonist -Indicated for acute postoperative and post-partum non-obstructive urine retention -May cause headache, bradycardia, diarrhea Adverse effects are rare. -Monitor for adverse effects and monitor patient for orthostatic hypotension

The nurse is assessing an older adult patient diagnosed with osteoarthritis. Which information provided by the patient requires follow-up?

Use of glucosamine and chondroitin daily Rationale:The use of glucosamine and chondroitin in clinical trials has shown mixed results. They have also been implicated as increasing the risk for bleeding, so the nurse would flag this information for further follow-up and discussion. Use of acetaminophen every 4-6 hours, stretching exercises for 10 minutes each day, and weekly massages would not require follow-up or discussion.

A patient with obstructive sleep apnea reports feeling claustrophobic when using a continuous positive airway pressure (CPAP) device. Which intervention should the nurse recommend to this patient?

Use relaxation exercises. Rationale:Relaxation exercises may be useful for reducing the sensation of claustrophobia that some patients experience when wearing a CPAP device. Proper use of a CPAP device requires the patient to wear the correct size mask and to ensure that the facial straps are tight enough to form a snug seal against the patient's face. Nasal sprays and humidifiers may be used to reduce drying of the nasal and oral mucous membranes.

A 65-year-old female patient has been recently diagnosed with osteoporosis. Which information should the nurse include in the teaching related to the patient's diagnosis?

Walk 30-40 minutes per day. Rationale:Walking is a weight-bearing exercise. The patient should be encouraged to walk 30-40 minutes per day, at least four times a week, to promote bone growth. It is not necessary to decrease dietary iron intake, increase dietary protein, or completely abstain from caffeine.

Which exercise should the nurse suggest to a patient with osteoarthritis?

Water aerobic activities Rationale:Water aerobic exercises provide cardiopulmonary exercise and strengthen core muscle groups while cushioning joint impact. This is an appropriate intervention for the nurse to recommend to the patient. Tennis increases impact in the feet and ankle and knee joints and would not be appropriate for this patient. Interval training may be acceptable to walk 5 kilometers, but jogging will place impact on the ankle and knee joints. Meditation is helpful for focusing mental and spiritual energy but does not involve activity of the knee joints.

The nurse suspects that a patient who reports joint stiffness and swelling of the finger joints may have osteoarthritis. Which diagnostic test should the nurse anticipate will be ordered?

X-ray of the joint Rationale:X-rays are the most commonly used diagnostic test for osteoarthritis (OA). An x-ray would reveal a narrowing of the space between the bones of the joint; however, this may not happen until significant damage has occurred. Joint fluid analysis for bacteria or uric acid crystals can help rule out other possibilities such as infection or gout but does not help to diagnose OA. MRIs or ultrasounds, not CT scans, may be used to provide more detailed images of the bones and soft tissues and are more sensitive to joint damage than an x-ray. A sedimentation rate is a nonspecific blood test used to check for inflammation in the body, not to diagnose OA.

After receiving information on sleep hygiene, the patient asks, "How will I know if this works?" Which response should the nurse make to this patient?

You will report improved quality and quantity of sleep." Rationale:Evidence that teaching on sleep hygiene has been effective would be the patient reporting improved quality and quantity of sleep. The patient should not awaken at all during the night, should maintain a consistent bedtime, and should utilize relaxation techniques 30-45 minutes prior to bedtime to help induce sleep.

Electrolytes

a charged ion capable of conducting electricity

fibromyalgia

a chronic disorder characterized by widespread musculoskeletal pain, fatigue, and tenderness in localized areas. -there is no cure, treatment is symptomatic

urinary hesitancy

a delay and difficulty in initiating voiding; often associated with dysuria

MRI (magnetic resonance imaging)

a diagnostic technique that uses magnetic fields and radio waves to produce a detailed image of the body's soft tissue and bones

delirium tremens (DTs)

a disorder characterized by a sad or despondant mood or loss of interest in usual activities

Narcolepsy

a disorder characterized by extreme daytime sleepiness with frequent episodes of "nodding off"

Sleep Apnea

a disorder characterized by frequent short breathing pauses during sleep

Foot Drop

a gait abnormality in which the dropping of the forefoot happens due to weakness

Joint Capsule

a ligamentous sac that surrounds the articular cavity of a freely movable joint. It is attached to the bones, completely encloses the joint, and is composed of an outer fibrous membrane and an inner synovial membrane.

osmolarity

a measure of the concentration of solutes in body fluids

Dependence

a physiologic need for a substance that the patient cannot control, and which results in withdrawl symptoms if the substance is witheld.

addiction

a psychological or physical need for a substance (such as alcohol) or process (such as gambling) to the extent that the individual will risk negative consequences in an attempt to meet the need

oncotic pressure

a pulling force exerted by colloids that helps maintain the water content of blood by pulling water from interstitial space into the vascular compartment

Isotonic

a solution that has the same osmolarity as body fluids

mobility

a state or quality of being mobile or immobile

creatinine clearance

a test uses 24-hour urine and serum creatinine levels to determine the glomerular filtration rate, a sensitive indicator of renal function

Cartilage

a type of flexible connective tissue found throughout the body -does not contain blood vessels

alcohol abuse

drinking too much alcohol, drinking it too often, or drinking it at inappropriate times

X-ray

electromagnetic radiation that differentially penetrates structures within the body and creates images of these structures on photographic film of fluorescent screen

urinary incontinence

involuntary urination due to the temporary or permanent inability of the external sphincter muscles to control the flow of urine from the bladder

What action will the nurse include in the plan of care for a patient who is being admitted with C. Diff?

isolate patient/contact precautions

When assessing a patient's first voided urine of the day, what finding should be reported to the HCP?

light pink urine

impaired physical mobility

limitation in independent, purposeful physical movement of the body or of one or more extremities

stress incontinence

loss of small amounts of urine when laughing, sneezing, or lifting primarily due to weak pelvic muscles, urethra, or surrounding tissues

Mineral oil

lubricant laxative -works by keeping water in the stool and intestines -indicated for treatment of constipation -may cause rectal leakage, irritation around anus

Gait

manner of walking

Laxatives

medications that stimulate bowel activity and assist fecal elimination

Osteoarthritis (OA)

most common form of arthritis in older adults; caused by chronic degenerative changes in the cartilage and synovial membranes of the joints. -part of the normal process of aging

muscle atrophy

muscle mass decreases in size

rapid eye movement (REM) sleep

occurs during sleep about every 90 minutes and lasts 5-30 minutes; the brain is highly active in this phase and most dreams will take place during REM sleep

Extracellular fluid volume excess

occurs when there is too much isotonic fluid in the extracellular compartment

A client seeking treatment for severe knee pain has worked in a factory for 30 years in a position requiring repetitive lifting and carrying of 20-40 pound boxes. Based on the clients history, the nurse should anticipate what initial recommendations from the multidisciplinary healthcare team?

pharmacologic therapy

Hypostatic pneumonia

pneumonia that usually results from the collection of fluid in the dorsal region of the lungs and occurs especially in those (as the bedridden or elderly) confined to a supine position for extended periods.

osmotic pressure

pressure that must be applied to prevent osmotic movement across a selectively permeable membrane

Oliguria

production of abnormally small amounts of urine by the kidney -Usually less than 500 mL/day or 30 mL/hr for an adult

Hypotonic

refers to solutions that have a lower osmolarity than body fluids, such as 1/2 normal saline

circadian rhythm

regular fluctuations in the body's physiologic processes occurring in a 24 hr cycle

suicide precautions

removal of harmful items; increased supervision to prevent acts of self-harm

Dulcolax (bisacodyl)

stimulant laxative -indicated for treatment of constipation, clean out intestines preoperatively -works by increasing movement of the intestines -may cause abdominal pain, cramping, nausea, diarrhea, weakness

Most common urinary incontinence in women

stress incontinence and overactive bladder

Colloids

substances such as large protein molecules that do not readily dissolve into true solutions

Buffers

substances that prevent major changes in pH by releasing hydrogen ions

Cataplexy

sudden loss of muscle control

Urgency

sudden strong desire to void

Insomnia

the inability to fall asleep

Center of Gravity

the point around which an object's weight is evenly distributed.

Flatulence

the presence of excessive amounts of gas in the stomach or intestines

body mechanics

the proper use of the body to facilitate lifting and moving and prevent injury

Elimination

the secretion and excretion of body wastes from the kidneys and intestines

Activity Tolerance

the type and amount of exercise or daily activities an individual is able to perform

Body Alignment

the way the head, trunk, arms, and legs are aligned with one another; posture

A nurse is talking with the adult daughter of an​ 80-year-old client who was recently discovered to be abusing prescription narcotics. The daughter expresses frustration that this substance abuse​ wasn't discovered​ sooner, and she asks the nurse how her​ father's previous healthcare providers could have overlooked this problem. Which of the following statements would not be appropriate for the nurse to include in her​ reply?

​"Substance abuse and addiction are almost unheard of among older​ adults, so few providers would consider the possibility of these diagnoses when working with clients like your​ father."


Related study sets

Ancient India (Indus River Valley)

View Set

farma uno - uždegimas, reumatas

View Set

Section 4: Financing Real Estate in Texas

View Set

Chapter 12 Development Through the Lifespan, 7e

View Set

World history Unit 2 Study Guide

View Set

Business Policy Test Chapters 7-10

View Set

Ch. 5 Job-Based Structures and Job Evaluation - MGT 461

View Set